$\sum_{a\lt n\le b}\phi (n)=\int_a^b \phi (x)\, dx+\int_a^b (x-[x]-\frac{1}{2})\phi '(x)\, dx+(a-[a]-\frac{1}{2})\phi (a)-(b-[b]-\frac{1}{2})\phi (b)$

Aug 15 2020

จากThe Theory of the Riemann Zeta-Functionของ Titchmarsh หน้า 13:

ปล่อย $\phi (x)$ เป็นฟังก์ชันใด ๆ ที่มีอนุพันธ์ต่อเนื่องในช่วงเวลา $[a,b]$. จากนั้นถ้า$[x]$ หมายถึงจำนวนเต็มมากที่สุดไม่เกิน $x$, $$\sum_{a\lt n\le b}\phi (n)=\int_a^b \phi (x)\, dx+\int_a^b \left(x-[x]-\frac{1}{2}\right)\phi '(x)\, dx+\left(a-[a]-\frac{1}{2}\right)\phi (a)-\left(b-[b]-\frac{1}{2}\right)\phi (b).$$

ไม่มีหลักฐานเรื่องนี้ในหนังสือและฉันไม่รู้ว่า 'ชื่อ' ของทฤษฎีบทนี้คืออะไร ฉันอยากเข้าใจทฤษฎีบทนี้ แต่ไม่รู้จะเริ่มจากตรงไหน

คำตอบ

3 OliverDiaz Aug 16 2020 at 00:01

ปล่อย $\rho(t)=\frac12 -(t-[t])=\frac{1}{2} - \{t\}$, ที่ไหน $\{t\}$ คือส่วนที่เป็นเศษส่วนของ $t$.

ร่างหลักฐาน:

ฉันฝากรายละเอียดให้คุณ นี่เป็นวิธีหนึ่งในการเข้าถึงตัวตนนี้

  • ก่อนอื่นให้สังเกตว่า $\rho$ คือ $1$- ฟังก์ชันต่อเนื่องและที่ $\rho'(t)=-1$ สำหรับ $x\in [k,k-1)$, $k\in\mathbb{Z}$. สำหรับ$k\leq \alpha<b\leq k+1$ใช้การรวมทีละส่วนสองครั้ง (ครั้งเดียวกับ $u=f(t)$ และ $dv=\rho'(t)\,dt$; และอื่น ๆ ด้วย$u=f'(t)$ และ $dv=\sigma'(t)\,dt=\rho(t)\,dt$) เพื่อรับ

$$ \begin{align} -\int^\beta_\alpha f(t)\,dt &= \int^\beta_\alpha f(t)\rho'(t)\,dt\\ &=\rho(\beta-)f(\beta)-\rho(\alpha)f(\alpha)-\int^\beta_\alpha \rho(t)\,f'(t)\,dt \end{align} $$

ตอนนี้คุณสามารถเพิ่มช่วงเวลาจำนวนเต็มได้ $[k,k+1]\subset(a,b]$ แล้วในช่วงเวลาเศษส่วน $(a,[a]+1]$, $[[b],b]$ เพื่อให้ได้ผลลัพธ์ที่ต้องการ


แก้ไข:สามารถรับหลักฐานทั่วไปและสง่างามได้จากการรวมตามส่วนต่างๆ:

เลม:ปล่อย$F$ และ $G$ เป็นฟังก์ชันต่อเนื่องที่ถูกต้องของรูปแบบ จำกัด เฉพาะที่เปิดอยู่ $I$และปล่อยให้ $\mu_G$, $\mu_F$ เป็นมาตรการลงนามที่เกิดจาก $G$ และ $F$ตามลำดับ จากนั้นสำหรับช่วงเวลาที่กะทัดรัด$[a,b]\subset I$, $$ \begin{align} \int_{(a,b]} F(t)\,\mu_G(dt)=F(b)G(b)-F(a)G(a)-\int_{(a,b]}G(t-)\,\mu_F(dt) \end{align} $$ ที่ไหน $G(t-)=\lim_{s\nearrow t}G(s)$.

สำหรับ OP

พิจารณาการวัดการนับ $\mu(dt)=\sum_{n\in\mathbb{Z}}\delta_{n}$ และมาตรการ Lebesgue $\lambda$ทั้งที่กำหนดไว้ใน $(\mathbb{R}\mathscr{B}(\mathbb{R}))$. ปล่อย$\phi(dt)=(\lambda-\mu)(dt)$. สังเกตว่า$\Phi(t):=\phi((0,t])=t-[t]=\{t\}$.

$$ \begin{align} \sum_{a< n\leq b}f(n)-\int^b_af(t)\,dt &=-\int^b_af(t)\,(\mu(dt)-\lambda(dt))=-\int^b_af(t)\phi(dt) \end{align} $$

ใช้ Lemma ด้านบนด้วย $f$ แทนที่ $F$ และ $\Phi$ แทนที่ $G$เรามีสิ่งนั้น $\mu_f(dt)=f'(t)\,dt$ และ $\mu_{\Phi}(dt)=\phi(dt)$ และอื่น ๆ

$$ \begin{align} \int^b_af(t)\phi(dt) &= f(t)\Phi(t)|^b_a -\int^b_a\Phi(t-)\, f'(t)\,dt\\ &=f(b)\{b\}-f(a)\{a\}-\int^b_a\Phi(t)\,f'(t)\,dt\\ &= f(b)(b-[b])-f(a)(a-[a)] -\int^b_a(t-[t])\,f'(t)\,dt \end{align} $$

การเปลี่ยนแปลงจากที่ไหน $\Phi(t-)$ ถึง $\Phi(t)$ ตามมาจากความจริงที่ว่า $\Phi(t-)=\Phi(t)$ $\lambda$-เช่น

ข้อสรุปตามด้วยการบวกและการลบ $\frac12$ ในอินทิกรัลสุดท้าย

1 Diger Aug 16 2020 at 03:48

นี่คือโดย Abel-Summation: $$\sum_{a<n\leq b} f(n) = f(b) \sum_{a<n\leq b} 1 - \int_a^b \sum_{a<n\leq t} 1 \cdot f'(t) \, {\rm d}t \\ = f(b) \left( \lfloor b \rfloor - \lfloor a \rfloor \right) - \int_a^b \left( \lfloor t \rfloor - \lfloor a \rfloor \right) f'(t) \, {\rm d}t \\ = f(b) \lfloor b \rfloor - f(a) \lfloor a \rfloor + \int_a^b \left(t - \lfloor t \rfloor - \frac{1}{2} + \frac{1}{2} - t \right) f'(t) \, {\rm d}t \\ = f(a) \left( a - \lfloor a \rfloor - \frac{1}{2} \right) - f(b) \left( b - \lfloor b \rfloor - \frac{1}{2} \right) + \int_a^b f(t) \, {\rm d}t + \int_a^b \left(t - \lfloor t \rfloor - \frac{1}{2} \right) f'(t) \, {\rm d}t \\ = f(a) \, B_1\left( a - \lfloor a \rfloor \right) - f(b) \, B_1\left( b - \lfloor b \rfloor \right) + \int_a^b f(t) \, {\rm d}t + \int_a^b B_1\left( t - \lfloor t \rfloor \right) f'(t) \, {\rm d}t \, ,$$ ที่ไหน $B_1(x)$เป็นพหุนามเบอร์นูลลีตัวแรก ดังที่ได้กล่าวไว้ก่อนหน้านี้$1/2$- เงื่อนไขซ้ำซ้อน

โดยการรวมอย่างต่อเนื่องโดยใช้ส่วนต่างๆ $\int B_n(x) \, {\rm d}x = \frac{B_{n+1}(x)}{n+1}$คุณจะได้รับสูตร Euler-Maclaurin หาก $a,b$ เป็นจำนวนเต็ม